You are on page 1of 9

Pharm D Exam

1. All of the following are incorporated into the Cockcroft–Gault equation except:
A. Age
B. Gender
C. Serum albumin
D. Serum creatinine
2. Polypharmacy use in older adults does not result in:
A. Increased adherence
B. Increased drug–drug interaction
C. Increased complex regimen
D. Increased hospitalization
3. According to the 2012 Beers criteria, the following drug should be avoided in older adults:
A. Diazepam
B. Warfarin
C. Aspirin
D. Pravastatin
4. The predictors of adverse drug reactions include the following except:
A. More than four medications
B. Longer than 14 days of hospital stay
C. More than four active medical problems
D. Smoking history
5. Medication nonadherence among older adults is influenced by:
A. More than two prescribers
B. Four or more medication changes in past 12 months
C. History of more than two surgeries
D. Having no caregiver help
6. The following statement about pharmacotherapy in older adults is false:
A. Renal function needs to be monitored for patients on digoxin.
B. Beers criteria indicate inappropriate medications.
C. Albumin needs to be monitored for patients on phenytoin.
D. Drug monitoring is often unnecessary due to multimorbidity
6. MM is a 6-month-old male infant who was born at 34-week GA. You are asked to evaluate his
renal function in preparation for starting intravenous antibiotics. Which method for assessment is
most appropriate?
A. Cockcroft–Gault equation
B. Schwartz (original) equation
C. Modification of diet in renal disease (MDRD) equation
D. Urine output alone
7. 68-year-old white man has resistant hypertension, prior myocardial infarction, and chronic kidney
disease (CKD; serum creatinine 1.8 mg/dL [159 μmol/L], estimated creatinine clearance 40 mL/min
[0.67 mL/s]). You are initiating ramipril today. What is the most appropriate timeframe for
laboratory follow-up?
A. 1 to 2 days
B. 1 to 2 weeks
C. 1 to 2 months
D. 3 to 4 months
8. A 67-year-old Asian man with a recent non-ST segment elevation MI (2 weeks ago) has an average
BP of 148/86 mm Hg and a heart rate of 76 beats/min. Which of the following antihypertensive
agents is preferred in this setting?
A. Metoprolol tartrate
B. Acebutolol
C. Hydrochlorothiazide
D. Spironolactone
9. Which of the following is TRUE regarding ACE inhibitors in heart failure?
A. Should be used mainly in severe heart failure, NYHA functional class IV
B. Efficacy of ACE inhibitors is a class effect
C. May be used in place of hydralazine and isosorbide dinitrate in cases of renal dysfunction
D. Can be replaced by angiotensin receptor blockers if the patient has hyperkalemia
10. A 74-year-old woman presents to clinic for heart failure follow-up. She is classified as NYHA FC II.
Her blood pressure is 144/82 mm Hg, and most recent EF is 26% (0.26). Her current medication
regimen includes lisinopril 20 mg Qday, carvedilol 25 mg BID, digoxin 0.125 mg Qday, and
furosemide 20 mg BID. Which of the following would be the BEST choice to add at this time?
A. Metolazone
B. Hydralazine and isosorbide
C. Spironolactone
E. Hydrochlorothiazide
11. Mineralocorticoid receptor antagonists (aldosterone receptor antagonists) have been shown to
reduce mortality in patients with heart failure. Which of the following is TRUE about MRAs?
A. Spironolactone leads more frequently to gynecomastia compared to eplerenone
B. Associated with hypokalemia
C. Can only be used in NYHA functional class IV
D. Used after maximizing ACE inhibitors, β-blockers, and digoxin
12. Which of the following diuretic combinations is used for the purpose of reducing congestion in the
setting of diuretic resistance?
A. Hydrochlorothiazide and spironolactone
B. Spironolactone and torsemide
C. Furosemide and spironolactone
D. Furosemide and metolazone
13. A 50-year-old, nonsmoking woman has no significant past medical history. A physical exam and
laboratory tests reveal the following: Height 5’4” (163 cm), weight 184 lb (83.6 kg), blood pressure
134/80 mm Hg, heart rate 70 beats/min, total cholesterol 184 mg/dL (4.76 mmol/L), LDL
cholesterol 110 mg/dL (2.84 mmol/L), HDL cholesterol 46 mg/dL (1.19 mmol/L), and triglycerides
140 mg/dL (1.58 mmol/L). Which of the following are risk factors for IHD in this patient?
A. Age, hypertension, dyslipidemia
B. Obesity
C. Age, dyslipidemia
D. Obesity, hypertension
14. A 65-year-old postmenopausal woman has a history of hypertension, dyslipidemia, and chronic
stable angina. Her current medications are atenolol 50 mg PO daily, simvastatin 40 mg PO at
bedtime, and SL nitroglycerin as needed. She has allergies/intolerances to aspirin (angioedema) and
enalapril (cough). Which of the following should be added to her drug regimen to reduce her risk
for cardiovascular events?
A. clopidogrel
B. dipyridamole
C. niacin
D. ticagrelor
15. A 68-year-old man with a history of hypertension, dyslipidemia, and chronic obstructive
pulmonary disease was recently diagnosed with chronic stable angina. His current medications are
chlorthalidone 25 mg PO daily, atorvastatin 40 mg PO at bedtime, salmeterol one inhalation every
12 hours, fluticasone MDI two puffs twice a day, and albuterol MDI one to two puffs every 4 hours
prn. His vital signs are a heart rate of 86 beats/min and blood pressure of 150/90 mm Hg. In
addition to sublingual nitroglycerin, what is the most appropriate change to his drug therapy?
A. Start propranolol
B. Start amlodipine
C. Start isosorbide mononitrate
D. Start verapamil
16. Which of the following is the most appropriate treatment for a pregnant patient (first trimester)
with a newly diagnosed acute PE?
A. Enoxaparin SC 1 mg/kg twice daily
B. Fondaparinux SC 2.5 mg daily
C. UFH SC 333 units/kg followed by 250 units/kg twice daily
D. Warfarin dose adjusted to achieve an INR goal of 2 to 3
17. Which of the following is the most appropriate initial treatment option in a patient with an acute
DVT and a documented history of heparin-induced thrombocytopenia (HIT) 7 months ago but no
history of prior VTE?
A. Clopidogrel
B. Dabigatran
C. Dalteparin
D. Warfarin
18. Which of the following statements accurately describes a potential advantage of the direct oral
anticoagulants (apixaban, dabigatran, rivaroxaban) over warfarin in the treatment of VTE?
A. More patients are able to tolerate the direct oral anticoagulants than warfarin.
B. Adherence with direct oral anticoagulants is 20%–30% better than adherence to warfarin.
C. When bleeding occurs, direct oral anticoagulants are more easily reversed than warfarin.
D. The onset of anticoagulant activity is more rapid with direct oral anticoagulants when
compared to warfarin.
19. A 38-year-old woman presents to your clinic today with new onset chest pain that occurs after
walking a few blocks. Her father had a heart attack at age 40. Her diet is low in saturated fats and
cholesterol. The patient is otherwise healthy and does not smoke. Her lipid panel today is: total
cholesterol 350 mg/dL (9.05 mmol/L), HDL cholesterol 40 mg/dL (1.03 mmol/L), and triglycerides
120 mg/dL (1.36 mmol/L). What is her non-HDL cholesterol and LDL cholesterol?
A. 310 mg/dL (8.02 mmol/L) and 286 mg/dL (7.40 mmol/L)
B. 390 mg/dL (10.09 mmol/L) and 334 mg/dL (8.64 mmol/L)
C. 310 mg/dL (8.02 mmol/L) and unable to calculate
D. 350 mg/dL (9.05 mmol/L) and 286 mg/dL (7.40 mmol/L)
20. What is this patient’s primary target (in question 19) for intervention?
A. Triglycerides
B. LDL cholesterol
C. Non-HDL cholesterol
D. Non-HDL and LDL cholesterol
21. According to the ACC/AHA guidelines, what intervention would be most appropriate for this patient
(in question 19)?
A. Start simvastatin 80 mg daily
B. Start prescription omega-3 fatty acid esters 4 g daily
C. Start ezetimibe 10 mg daily
D. Start a high-intensity statin
22. According to the NLA, which risk category does this patient best fit?
A. Very High Risk
B. High Risk
C. Moderate Risk
D. Low Risk
23. Which side effect can be caused by statin therapy?
A. Fatty liver
B. New onset diabetes
C. Flushing
D. None of the above
24. A 67-year-old man with chronic cough and sputum production, an FEV1/FVC of 68% (0.68), and
an FEV1 40% of predicted can be classified according to the GOLD guidelines as:
A. Not having COPD
B. GOLD 1: mild
C. GOLD 2: moderate
D. GOLD 3: severe
25. A 59-year-old man presents with cough, sputum production, and dyspnea with exertion that began
6 months ago. He is a nonsmoker, but he has worked as a bartender four nights a week for the last
32 years. Lung examination reveals significant wheezing bilaterally. Which one of the following
confirms the suspected diagnosis of COPD?
A. PaCO2 55 mm Hg (7.3 kPa) on arterial blood gas
B. Exposure to secondhand smoke
C. Postbronchodilator FEV1/FVC ratio of 60% (0.60)
D. FEV1 75%
26. Which of the following PPI regimens is/are preferred as initial treatment in a 60-year-old patient
with erosive esophagitis?
A. Lansoprazole 30 mg once daily × 3 weeks
B. Pantoprazole 40 mg twice daily × 24 weeks
C. Esomeprazole 20 mg once daily × 8 weeks
D. Rabeprazole 20 mg once daily as needed
27. Which of the following risk factors would not be a justification for using a PPI in a patient on
clopidogrel?
A. Prior history of upper GI bleed
B. Advanced age
C. Erosive esophagitis
D. Intermittent GERD symptoms
28. Which recommendation would be most appropriate regarding calcium supplementation for
prevention of bone fractures in a healthy 24-year-old woman taking omeprazole 20 mg once daily
for GERD?
A. Elemental calcium 2000 mg daily in divided doses
B. No calcium is needed because she does not have risk factors for osteoporosis or fractures
C. Elemental calcium 2000 mg daily plus vitamin D 800 units daily
D. Calcium citrate 500 mg four times daily
29. PPIs decrease stomach acid by which of the following mechanisms?
A. Inhibiting gastric H+/K+-adenosine triphosphatase in gastric parietal cells
B. Inhibiting histamine2 receptors in gastric parietal cells
C. Forming a viscous solution that floats on the surface of the gastric contents
D. Forming a protective coating over the damaged mucosa
30. A 65-year-old woman presents with new onset epigastric pain, recent 10-pound (4.5kg) weight
loss, and anemia. What diagnostic test should this patient undergo?
A. Urea breath test
B. Stool antigen testing for H. pylori
C. Esophagogastroduodenoscopy (EGD)
D. H. pylori serology testing
31. Which one of the following should be considered when evaluating a patient who has failed H.
pylori eradication therapy?
A. Patient adherence
B. Preexisting antimicrobial resistance
C. Potential reinfection
D. All of the above
32. Loperamide should be avoided in patients with active UC due to its ability to induce which one of
the following complications?
A. Toxic megacolon
B. Intestinal stricture
C. Fistula formation
D. Pancreatitis
33. Probiotics may be most helpful in patients with IBD who have:
A. Pouchitis
B. Fistulae
C. Proctitis
D. Arthritis
34. Which one of the following medications would be contraindicated in a pregnant patient with CD?
A. Sulfasalazine
B. Budesonide
C. Infliximab
D. Methotrexate
35. Which one of the following disorders is a possible extraintestinal manifestation of inflammatory
bowel disease?
A. Erythema nodosum
B. Migraine headaches
C. Asthma
D. Sinusitis
36. Certolizumab works by inhibiting the activity of:
A. Interleukin-6
B. Leukocyte adhesion molecules
C. Tumor necrosis factor alpha
D. B lymphocytes
37. Which condition is considered a contraindication to infliximab therapy?
A. Diabetes
B. Hypothyroidism
C. Dyslipidemia
D. Advanced heart failure
38. How should an SAAG of 1.3 g/dL (13 g/L) be interpreted?
A. Indicates peritoneal infection as cause of ascites
B. Indicates portal hypertension as cause of ascites
C. Indicates heart failure as cause of ascites
D. Indicates malignancy as cause of ascites
39. CY is a 42-year-old Asian woman with the following laboratory values: eGFR 48 mL/min/1.73 m2
(0.46 mL/s/m2) and urine albumin:creatinine ratio 15 mg/g creatinine (1.7 mg/mmol creatinine).
How would you classify her CKD?
A. GFR category 2; Albuminuria category A1
B. GFR category 3a; Albuminuria category A1
C. GFR category 3b; Albuminuria category A2
D. GFR category 4; Albuminuria category A2
40. RK is a 75-year-old man who presents to his neurologist with symptoms of dementia of the
Alzheimer type. The neurologist decides to begin donepezil at 10 mg daily. Which of the following
should be of concern to the physician?
A. Tremors
B. Creatinine clearance
C. Agitation
D. Nausea
41. 35-year-old woman is being started on lamotrigine for newly diagnosed epilepsy. She is otherwise
healthy and is not taking any medications except for acetaminophen for an occasional headache.
Which of the following will be most important to monitor as she starts lamotrigine?
A. Vision
B. Blood count
C. Rash
D. Heart rhythm
42. A new patient with Parkinson disease comes to the pharmacy with a prescription for pramipexole
0.25 mg at bedtime. Which of the following is the most common adverse effect of pramipexole?
A. Obsessive behavior
B. Livedo reticularis
C. Nausea
D. Thickening of heart valves
43. What is the target TSH range (mIU/L or μIU/mL) for patients being treated for hypothyroidism or
hyperthyroidism?
A. Undetectable
B. 2.5 to 4.5
C. 1.4 to 2.5
D. 0.5 to 4
44. Untreated hyperthyroidism in the elderly can result in:
A. Mania
B. Atrial fibrillation
C. Deafness
D. Hirsutism
45. Potential risks associated with the use of oral contraceptives include all of the following except:
A. Venous thromboembolism
B. Hypertension
C. Gallbladder disease
D. Breast cancer
46. Which of the following COCs is FDA-approved for the treatment of PMDD?
A. Yaz
B. Yasmin
C. Seasonique
D. Loestrin 24 Fe
47. Unlike COCs, which of the following risks has not been associated with progestin-only pills?
A. Hypertension
B. Venous thromboembolism
C. Headaches
D. Acne
48. Use of a 5α-reductase inhibitor can be expected to reduce the volume of an enlarged prostate gland
by this percentage.
A. 10
B. 20
C. 50
D. 75
49. According to Hepler & Strand, the three overarching responsibilities of a pharmacist practicing
pharmaceutical care is to identify, resolve, and prevent which of the following?
A. Health insurance issues
B. Malpractice
C. Drug Related Problems
D. Legal obstacles to health care access
50. A pharmacist notices that his patient's prescription for lovastatin is unusually low and worries
that this low dosage may not be adequate enough to treat the patient's condition. Into which
category of drug therapy problems would this situation fall?
A. Adherence & convenience
B. Safety
C. Indication (appropriateness)
D. Effectiveness

You might also like